« Intégration de Riemann/Intégrales généralisées » : différence entre les versions

Contenu supprimé Contenu ajouté
modification exemple d'introduction
m màj+meftypo+rectif d'une bourde (variable d'intégration = même lettre que la borne)
Ligne 1 :
{{Chapitre
| idfaculté = mathématiques
| numéro = 65
| précédent = [[../MomentCentre d'inertie/]]
| suivant = [[../|Sommaire]]
| niveau = 14
}}
 
L'objectif de ce cours est d'apprendre à étudier la convergence (et éventuellement à faire le calcul) d'intégrales dont une borne est infinie comme :<br />
:<math>\int_1^{+\infty} \frac{\mathrm{d}x}{x^2}</math> <br />
ou encore avec au moins une borne où la fonction n’est pas définie et a une limite infinie comme :<br />
:<math>\int_0^{\frac{\pi}{2}} \tan x \,\mathrm{d}x dx</math> .
 
== Définitions et premières propriétés ==
=== Définition ===
On suppose dans la définition suivante (et même dans toute la suite) que le seul "problème" est sur la borne <math>b</math> (on procéderait de même en cas de problème sur la borne d’en bas) :<br />
 
{{Définition
| titre = Définition : Intégrale généralisée (ou impropre)
| contenu =
Soit <math>f</math> une fonction définie et continue par morceaux sur un intervalle <math>]a;b[</math> avec <math>a,b \in \R\cup \{\pm \infty\}</math> .<br />
 
On appelle '''intégrale généralisée de <math>f</math> entre <math>a</math> et <math>b</math> ''' la limite suivante :<br />
On appelle '''intégrale généralisée de <math>f</math> entre <math>a</math> et <math>b</math> ''' la limite suivante :
<center>
{{Résultat
| <math>\int_a^b f(t)\mathrm{d}t = \lim_{x\to b} \int_a^x f(t)\mathrm{d}t</math>.
}}
</center>
Ligne 29 ⟶ 31 :
}}
 
Remarquez que le symbole <math>\int_a^b f(t)\,\mathrm{d}t</math> n'a pas de sens si l'intégrale est divergente ou si l’on n'a pas prouvé sa convergence.<br />
 
'''Exemples :'''
 
1/ Montrer que <math>\int_0^1 \frac{\ln t}t\,\mathrm{d}t</math> diverge.
 
On remarque que <math>\int \frac{\ln t}t\,\mathrm dt = \frac12\ln^2 t</math>.
 
Alors : <math>\lim_{x\to 0} \int_x^1 \frac{\ln t}t\,\mathrm dt = \lim_{x\to0}\frac12(\ln^21-\ln^2x) = +\infty</math> donc l'intégrale diverge.
 
2/ Montrer que <math>\int_0^1 \ln t \,\mathrm dt</math> converge.
 
On remarque que <math>\int \ln t \,\mathrm{d}t = t\ln t - t</math>.
 
Alors : <math>\lim_{x\to 0} \int_x^1 \ln t \,\mathrm dt = \lim_{x\to 0} \left((1\ln 1 -1) - (x\ln x - x)\right) =\lim_{x\to 0} x - x\ln x - 1 = -1 </math> donc l'intégrale converge.
'''Exemples :''' <br />
1/ Montrer que <math>\int_0^1 \frac{\ln t}{t} \mathrm{d}t</math> diverge.<br />
On remarque que <math>\int \frac{\ln t}{t}\mathrm{d}t = \frac{1}{2}\ln ^2 t</math> .<br />
Alors : <math>\lim_{x\to 0} \int_x^1 \frac{\ln t}{t} \mathrm{d}t = \lim_{x\to 0} \frac{1}{2}(\ln ^2 1 - \ln ^2 x) = +\infty</math> donc l'intégrale diverge.<br />
2/ Montrer que <math>\int_0^1 \ln t \,\mathrm{d}t</math> converge.<br />
On remarque que <math>\int \ln t \,\mathrm{d}t = t\ln t - t</math> .<br />
Alors : <math>\lim_{x\to 0} \int_x^1 \ln t \,\mathrm{d}t = \lim_{x\to 0} \left((1\ln 1 -1) - (x\ln x - x)\right) =\lim_{x\to 0} x - x\ln x - 1 = -1 </math> donc l'intégrale converge.<br />
 
=== Premières propriétés ===
Lorsqu’il y a un problème sur les deux bornes, on utilise la Relationrelation de Chasles sur les intégrales généralisées '''convergentes''' :<br />
 
{{Propriété
| titre = Relation de Chasles sur les intégrales généralisées convergentes
| contenu =
Soit <math>f</math> une fonction continue par morceaux sur <math>]a;,b[</math> et <math>c\in ]a;,b[</math> .
Alors (sous réserve d'existence) :<br />
<center>
{{Résultat
| <math>\int_a^b f(t)\,\mathrm{d}t dt= \int_a^c f(t)\,\mathrm{d}t dt+ \int_c^b f(t) \,\mathrm{d}t dt</math>.
}}
</center>
}}
 
Il y a aussi linéarité des intégrales généralisées convergentes. <br />
 
Cela se démontre en utilisant les propriétés des intégrales et en passant à la limite.<br />
Cela se démontre en utilisant les propriétés des intégrales et en passant à la limite.
 
<u>Remarque :</u> Il faut "couper" pour connaître la nature d’une intégrale généralisée.
<br />Par exemple, on a :
<math>\int_{-x}^x \sin t \,\mathrm{d}t dt= 0 \;\forall x \in\R</math> converge et pourtant
<math>\int_{-\infty}^{+\infty} \sin t \,\mathrm{d}t dt</math> diverge (<math>x \mapsto -\cos x</math> est une primitive de <math> x\mapsto \sin x</math> et n'a pas de limite en l'infini).
 
Enfin, il y a les "fausses intégrales généralisées", celles où l’on règle le problème par '''prolongement par continuité''' de la fonction à intégrer :
 
Enfin, il y a les "fausses intégrales généralisées", celles où l’on règle le problème par '''prolongement par continuité''' de la fonction à intégrer :<br />
{{Exemple
| contenu =
:<math>\int_0^1 \frac{\sin t}{t} \,\mathrm{d}t dt</math> est convergente. <br />
Il suffit de remarquer que si <math>f : x\mapsto \frac{\sin x}{x}</math>, alors son prolongement par continuité en <math>0</math> est :<br />
:<math>g : x \mapsto \begin{cases} \frac{\sin x}{x}, & \mbox{si } x \ne 0 \\ 1 , & \mbox{si } x = 0 .\end{cases}</math>.
}}
 
Ligne 75 ⟶ 86 :
{{Exemple
| titre = Exemple de Riemann
| contenu = Soit <math>\alpha\in \R_+^*</math> .<br />
* <math>\int_1^{+\infty} \frac{\mathrm{d}t dt}{t^{\alpha}}</math> converge si, et seulement si <math>\alpha > 1</math> .
* <math>\int_0^1 \frac{\mathrm{d}t dt}{t^{\alpha}}</math> converge si, et seulement si <math>\alpha < 1</math> .}}
 
{{Démonstration déroulante
| contenu = On démontre seulement le résultat en <math>+\infty</math> .<br />
 
Il suffit de revenir à la définition d'intégrale généralisée comme limite :<br />
Il suffit de revenir à la définition d'intégrale généralisée comme limite :
 
* '''Cas où <math>\alpha \ne 1</math> ''' :
<math>I_{\alpha} = \int_1^{+\infty} \frac{\mathrm{d}t}{t^{\alpha}} =
\left[\frac{t^{1-\alpha}}{1-\alpha}\right]_1^{+\infty} = \lim_{x\to +\infty}\frac{1}frac1{1-\alpha}(x^{1-\alpha} - 1)</math>.<br />
 
Donc :<br />
Donc :
- si <math>\alpha > 1</math> , alors <math>1-\alpha < 0 \Rightarrow \lim_{x\to +\infty} x^{1-\alpha} = 0</math> donc <math>I_{\alpha}</math> converge ;<br />
 
- si <math>\alpha < 1</math> , alors <math>1-\alpha > 0 \Rightarrow \lim_{x\to +\infty} x^{1-\alpha} = +\infty</math> donc <math>I_{\alpha}</math> diverge .<br />
- si <math>\alpha > 1</math> , alors <math>1-\alpha < 0 \Rightarrow \lim_{x\to +\infty} x^{1-\alpha} = 0</math> donc <math>I_{\alpha}</math> converge ;
 
- si <math>\alpha < 1</math> , alors <math>1-\alpha > 0 \Rightarrow \lim_{x\to +\infty} x^{1-\alpha} = +\infty</math> donc <math>I_{\alpha}</math> diverge.
 
* '''Cas où <math>\alpha = 1</math>''' :
On a alors <math>I_{1} I_1= \int_1^{+\infty} \frac{\mathrm{d dt}t}{t} =
\left[\ln t\right]_1^{+\infty} = \lim_{x\to +\infty}\ln x </math>.<br />
 
Il est clair que <math>I_1</math> diverge.
}}
 
== Convergence absolue et Théorèmethéorème de comparaison ==
 
=== Théorème de comparaison pour les intégrales généralisées ===
On considère dans tout ce paragraphe des fonctions '''à valeurs positives'''.<br />
 
{{Lemme
| contenu =
Soit <math>f>0</math> continue par morceaux sur <math>[a;,b[</math>.
<math>\int_a^b f(t)\,\mathrm{d}t dt</math> converge si, et seulement si, la fonction <math>F : x\mapsto \int_a^x f(t)\,\mathrm{d}t dt</math> est majorée sur <math>[a;,b[</math> .
}}
 
{{Démonstration déroulante
| contenu =
<math>F</math> est une primitive de <math>f</math> et <math>f = F'>0</math> donc <math>F</math> est croissante (et majorée).<br />
 
Le [[Fonctions d'une variable réelle/Limites|Théorème de la limite monotone]] permet alors de conclure.
Le [[Fonctions d'une variable réelle/Limites|théorème de la limite monotone]] permet alors de conclure.
}}
 
Voici maintenant le Théorèmethéorème central du cours :
 
{{Théorème
| titre = Théorème de comparaison (intégrales généralisées)
| contenu =
Soient <math>f</math> et <math>g</math> deux fonctions continues par morceaux et '''positives''' sur <math>[a;,b[</math> et telles que <math>f\le g</math>.<br />
* Si <math>\int_a^b g(t)\,\mathrm{d}t dt</math> converge, alors <math>\int_a^b f(t)\,\mathrm{d}t dt</math> converge aussi.
* Si <math>\int_a^b f(t)\,\mathrm{d}t dt</math> diverge, alors <math>\int_a^b g(t)\,\mathrm{d}t dt</math> diverge aussi.
}}
 
{{Démonstration déroulante
| contenu =
Le deuxième résultat est la contraposée du premier.<br />
 
Soient <math>F : x \mapsto \int_a^x f(t)\mathrm{d}t</math> et <math>G : x \mapsto \int_a^x g(t)\mathrm{d}t</math>.<br />
Soient <math>F : x \mapsto \int_a^x f(t)\,\mathrm dt</math> et <math>G : x \mapsto \int_a^x g(t)\,\mathrm dt</math>.

Par comparaison d'intégrales, <math>f\le g \Rightarrow F\le G\;(\star)</math> .Donc si <math>\int_a^b g(t)\,\mathrm{d}t dt</math> converge, alors <math>G</math> converge et est donc majorée(d'après le Lemmelemme), ce qui implique d’après <math>(\star)</math> que <math>F</math> aussi et donc (toujours grâce au Lemmelemme) que <math>\int_a^b f(t)\,\mathrm{d}t dt</math> converge.
}}
 
'''Exemple :'''<br />
 
Montrer que <math>\int_1^{+\infty} \mathrm{e}^{-t^2} \;\mathrm{d}t</math> converge.<br />
Montrer que <math>\int_1^{+\infty} \mathrm e^{-t^2} \;\mathrm dt</math> converge.
 
On remarque que :
<math>\begin{align}
\forall t \ge 1, t^2 \ge t &\Rightarrow -t^2 \le -t\\
&\Rightarrow 0\le \mathrm{ e}^{-t^2} \le \mathrm{ e}^{-t}\\
&\Rightarrow 0\le \int_1^x \mathrm{ e}^{-t^2} \mathrm{d}t dt \le \int_1^x \mathrm{ e}^{-t}\,\mathrm{d}t dt.
\end{align}</math>.
 
Mais <math>\int_1^x \mathrm{ e}^{-t}\,\mathrm{d}t dt= [-\mathrm{ e}^{-t}]_1^x = \frac{1}frac1{\mathrm{ e}} - \mathrm{ e}^{-x} \xrightarrow[x\to +\infty]{} \frac{1}frac1{\mathrm{ e}}</math> donc <math>\int_1^{+\infty} \mathrm{ e}^{-t} \;\mathrm{d}t dt</math> converge et <math>\int_1^{+\infty} \mathrm{ e}^{-t^2} \;\mathrm{d}t dt</math> converge aussi.
 
On rappelle que le "problème" est sur la borne d’en haut <math>b</math> (c'est donc en <math>b</math> que l’on effectue la comparaison de <math>f</math> et <math>g</math> ) :
 
{{Corollaire
| titre = Corollaire : Intégration des relations de comparaison
| contenu =
Soient <math>f</math> et <math>g</math> deux fonctions continues par morceaux et '''positives''' sur <math>[a;,b[</math>.<br />
 
'''1/''' On suppose que <math>f \underset{b}{=} O(g)</math><br />
'''1/''' On suppose que <math>f \underset{b}{=} O(g)</math>.
* Si <math>\int_a^b g(t)\mathrm{d}t</math> converge, alors <math>\int_a^b f(t)\mathrm{d}t</math> converge aussi.
* Si <math>\int_a^b fg(t)\,\mathrm{d}t dt</math> divergeconverge, alors <math>\int_a^b gf(t)\,\mathrm{d}t dt</math> divergeconverge aussi.
'''2/'''* Si <math>f\underset{b}{\sim} g</math> , alors les intégrales <math>\int_a^b f(t)\,\mathrm{d}t dt</math> etdiverge, alors <math>\int_a^b g(t)\,\mathrm{d}t dt</math> ontdiverge même nature (soit elles sont toutes les deux convergentes, soit elles sont toutes les deux divergentes)aussi.
'''2/''' Si <math>f\underset b{\sim} g</math> , alors les intégrales <math>\int_a^b f(t)\,\mathrm dt</math> et <math>\int_a^b g(t)\,\mathrm dt</math> sont de même nature (soit toutes les deux convergentes, soit toutes les deux divergentes).
}}
 
Pour un rappel sur les relations de comparaison, voyez [[Fonctions d'une variable réelle/Relations de comparaison]].
{{Démonstration déroulante|contenu =
'''1/''' Il suffit d’utiliser la définition de <math>f \underset{ b}{=} O(g)</math> : si <math>f = \varphi g</math> , alors <math>\exist M>0 | \quad\forall x \in [a;,b[ \;\varphi(x) \le M \Rightarrow \exist M>0|\quad\forall x \in [a;,b[ \;f(x) \le Mg(x) \;(1)</math> .<br />
 
L'inégalité <math>(1)</math> et le Théorème de comparaison permettent de conclure.<br />
L'inégalité <math>(1)</math> et le théorème de comparaison permettent de conclure.
'''2/''' On remarquera que <math>f\underset{b}{\sim} g \iff f \underset{b}{=} O(g) \mathrm{\;et\;} g \underset{b}{=} O(f)</math>.On utilise alors le point précédent.<br />
 
'''2/''' On remarquera que <math>f\underset b{\sim} g \iff f \underset b{=} O(g) \text{ et } g \underset{b}{=} O(f)</math>. On utilise alors le point précédent.
 
<u> Remarque :</u> Pour obtenir <math>(1)</math> , on a multiplié les deux membres par <math>g(x)</math> dans l'inégalité <math>\varphi(x) \ge M</math>. C'est possible car la fonction <math>g</math> est positive. }}
 
'''Exemple :''' Montrer que <math>\int_2^{+\infty} \frac{\mathrm{d}t dt}{\sqrt{t^4-1}}</math> converge.<br />
 
On remarque que <math>\frac{1}{\sqrt{t^4-1}} \;\underset{+\infty}{\sim}\; \frac{1}{t^2}</math> . L'exemple de Riemann permet alors de conclure.
On remarque que <math>\frac1{\sqrt{t^4-1}} \;\underset{+\infty}{\sim}\; \frac1{t^2}</math>. L'exemple de Riemann permet alors de conclure.
 
Mais que faire pour des fonctions qui ne sont pas nécessairement positives ? Il faudra souvent tenter d’utiliser la convergence absolue.
Ligne 168 ⟶ 196 :
| titre = Définition : Convergence absolue
| contenu =
Soit <math>f\in \mathcal{CM}([a;,b[)</math> .<br />
 
L'intégrale <math>\int_a^b f(t)\mathrm{d}t</math> est dite '''absolument convergente''' si, et seulement si, l'intégrale <math>\int_a^b |f(t)|\mathrm{d}t</math> converge.
L'intégrale <math>\int_a^b f(t)\,\mathrm dt</math> est dite '''absolument convergente''' si l'intégrale <math>\int_a^b |f(t)|\,\mathrm dt</math> converge.
}}
 
Ligne 176 ⟶ 205 :
}}
 
{{Démonstration déroulante|contenu = Soit <math>x\in [a;,b[</math> .<br />
On [[Intégration (mathématiques)/Propriétés de l'intégrale|a montré]] que :<br />
<center><math>\forall f\in\mathcal{CM}([a;b[),\, 0 \le \left|\int_a^x f(x)\mathrm{d}x \right| \le \int_a^x |f(x)|\mathrm{d}x</math> </center>
Le Théorème de comparaison permet de conclure.}}
 
On [[Intégration (mathématiques)/Propriétés de l'intégrale|a montré]] que :
Remarquez que la réciproque est fausse : on parle alors de '''semi-convergence'''.<br />
<center><math>\forall f\in\mathcal{CM}([a,b[),\, 0 \le \left|\int_a^x f(t)\,\mathrm dt\right| \le \int_a^x |f(t)|\,\mathrm dt</math>.</center>
'''Exemple :''' Montrer que l'intégrale <math>\int_1^{+\infty} e^{-x} \sin x \;\mathrm{d}x</math> est absolument convergente.<br />
Le théorème de comparaison permet de conclure.}}
Il suffit de remarquer que <math>|\sin x| \le 1 \; \forall x\in \R \Rightarrow |e^{-x} \sin x |\le e^{-x}</math> .<br />
 
On a montré au paragraphe précédent que <math>\int_1^{+\infty} e^{-x} \;\mathrm{d}x</math> converge . Le Théorème de comparaison permet de conclure.<br />
Remarquez que la réciproque est fausse : on parle alors de '''semi-convergence'''.
 
'''Exemple :''' Montrer que l'intégrale <math>\int_1^{+\infty}\mathrm e^{-x} \sin x \;\mathrm dx</math> est absolument convergente.
 
Il suffit de remarquer que <math>|\sin x| \le 1 \; \forall x\in \R \Rightarrow |\mathrm e^{-x} \sin x |\le\mathrm e^{-x}</math>.
 
On a montré au paragraphe précédent que <math>\int_1^{+\infty}\mathrm e^{-x} \;\mathrm dx</math> converge. Le théorème de comparaison permet de conclure.
 
== Intégration par parties et changement de variables ==
 
On peut utiliser une intégration par parties ou un changement de variables comme pour des intégrales "habituelles" (voyez [[Intégration (mathématiques)/Intégrale et primitives]]) ''' à la condition expresse de n'utiliser que des intégrales convergentes'''.<br />
 
'''Exemple :'''
 
Calculer <math>\int_1^{+\infty} \frac{\ln t}{t^2}\;\mathrm dt</math>.
 
On intègre par parties en posant :
:<math> u'(t) = \frac1{t^2} \Rightarrow u(t) = -\frac1t</math>
:<math> v(t) = \ln t \Rightarrow v'(t) = \frac1t</math>
donc <math>\forall x > 1 </math> :
:<math>\int_1^x\frac{\ln t}{t^2}\;\mathrm dt = \left[ -\frac{\ln t}t\right]_1^x+ \int_1^x \frac{\mathrm dt}{t^2}</math>.
 
On "passe à la limite" et l'on obtient :
'''Exemple :''' <br />
Calculer :<math>\int_1^{+\infty} \frac{\ln t}{t^2}\;\mathrm dt = \lim_{dx\to +\infty}t (-\frac{\ln x}x-\frac1x+ 1)</math>.<br />
On intègre par parties en posant :<br />
<math> u'(t) = \frac{1}{t^2} \Rightarrow u(t) = -\frac{1}{t}</math><br />
<math> v(t) = \ln t \Rightarrow v'(t) = \frac{1}{t}</math><br />
donc <math>\forall x > 1 </math>:<br />
<math>\int_1^{x} \frac{\ln t}{t^2}\;\mathrm{d}t = \left[ -\frac{\ln t}{t}\right]_1^{x} + \int_1^x \frac{\mathrm{d}t}{t^2}</math><br />
On "passe à la limite" et on obtient :<br />
<math>\int_1^{+\infty} \frac{\ln t}{t^2}\;\mathrm{d}t = \lim_{x\to +\infty} (-\frac{\ln x}{x} - \frac{1}{x} + 1)</math><br />
soit :<br />
<center>{{Encadre|contenu=<math>\int_1^{+\infty} \frac{\ln t}{t^2}\;\mathrm{d}t = 1</math>}}</center>.
 
soit :
<center>{{Encadre|contenu=<math>\int_1^{+\infty} \frac{\ln t}{t^2}\;\mathrm dt = 1</math>.}}</center>
 
{{Bas de page
| idfaculté = mathématiques
| précédent = [[../MomentCentre d'inertie/]]
| suivant = [[../|Sommaire]]
}}